Med Surg Final

Lakukan tugas rumah & ujian kamu dengan baik sekarang menggunakan Quizwiz!

After a vaginoplasty, what instruction by the nurse is most important? a. Avoid vaginal douching to prevent infection. b. Do not have sexual intercourse for at least 6 months. c. Use oil-based lubricants with the vaginal dilators. d. You must dilate the vagina several times a day for months.

d. You must dilate the vagina several times a day for months.

A 25-year-old patient diagnosed with invasive cervical cancer expresses a desire to have children. What procedure might the physician offer as treatment? a. Radical hysterectomy b.Radical culposcopy c. Radical trabeculectomy d.Radical trachelectomy

d.Radical trachelectomy

A patient has just returned to the floor following a transurethral resection of the prostate. A triple-lumen indwelling urinary catheter has been inserted for continuous bladder irrigation. What, in addition to balloon inflation, are the functions of the three lumens? A) Continuous inflow and outflow of irrigation solution B) Intermittent inflow and continuous outflow of irrigation solution C) Continuous inflow and intermittent outflow of irrigation solution D) Intermittent flow of irrigation solution and prevention of hemorrhage

A) Continuous inflow and outflow of irrigation solution

A patient who has had a lumpectomy calls the clinic to talk to the nurse. The patient tells the nurse that she has developed a tender area on her breast that is red and warm and looks like someone drew a line with a red marker. What would the nurse suspect is the womans problem? A) Mondor disease B) Deep vein thrombosis (DVT) of the breast C) Recurrent malignancy D) An area of fat necrosis

A) Mondor disease

A patient has had a total mastectomy with immediate reconstruction. The patient asks the nurse when she can take a shower. What should the nurse respond? A) Not until the drain is removed B) On the second postoperative day C) Now, if you wash gently with soap and water D) Seven days after your surgery

A) Not until the drain is removed

The nurse is discharging a patient home from an outpatient surgery center. The nurse has reviewed all of the discharge instructions with the patient and her caregiver. What else should the nurse do before discharging the patient from the facility? Select all that apply. A) Provide all discharge instructions in writing. B) Provide the nurses or surgeons contact information. C) Give prescriptions to the patient. D) Irrigate the patients incision and perform a sterile dressing change. E) Administer a bolus dose of an opioid analgesic.

A) Provide all discharge instructions in writing. B) Provide the nurses or surgeons contact information. C) Give prescriptions to the patient.

The nurse is leading a workshop on sexual health for men. The nurse should teach participants that organic causes of erectile dysfunction include what? Select all that apply. A) Diabetes B) Testosterone deficiency c. Anxiety d. Depression e. Parkinsonism

A) Diabetes B) Testosterone deficiency e. Parkinsonism

A patient who is in the first trimester of pregnancy has experienced an incomplete abortion. The obstetric nurse should prepare the patient for what possible intervention? A) Dilation and evacuation B) Several days of bed rest C) Administration of hydromorphone D) IV administration of clomiphene

A) Dilation and evacuation

A couple has come to the infertility clinic because they have been unable to get pregnant even though they have been trying for over a year. Diagnostic tests are planned for the woman to ascertain if ovulation is regular and whether her endometrium is adequately supported for implantation. What test would the nurse expect to have ordered for this woman? A) Serum progesterone B) Abdominal CT C) Oocyte viability test D) Urine testosterone

A) Serum progesterone

The nurse is working with a couple who have been unable to conceive despite more than 2 years of trying to get pregnant. The couple has just learned that in vitro fertilization (IVF) was unsuccessful and they are both tearful. What nursing diagnosis is most likely to apply to this couple? A) Hopelessness related to failed IVF B) Acute confusion related to reasons for failed IVF C) Compromised family coping related to unsuccessful IVF D) Moral distress related to unsuccessful IVF

A) Hopelessness related to failed IVF

A nurse is collecting assessment data from a premenopausal patient who states that she does not have menses. What term should the nurse use to document the absence of menstrual flow? A) Amenorrhea B) Dysmenorrhea C) Menorrhagia D) Metrorrhagia

A) Amenorrhea

A patient has just returned to the unit from the PACU after surgery for a tumor within the spine. The patient complains of pain. When positioning the patient for comfort and to reduce injury to the surgical site, the nurse will position to patient in what position? A) In the high Fowlers position B) In a flat side-lying position C) In the Trendelenberg position D) In the reverse Trendelenberg position

B) In a flat side-lying position

A premenopausal patient is complaining of vaginal spotting and sharp, colicky lower abdominal pain. She informs the nurse that her period is 2 weeks late. The nurse should recognize a need for this patient to be investigated for what health problem? A) Trichomonas vaginalis B) Ectopic pregnancy C) Cervical cancer D) Fibromyalgia

B) Ectopic pregnancy

A patient with ovarian cancer is admitted to the hospital for surgery and the nurse is completing the patients health history. What clinical manifestation would the nurse expect to assess? A) Fish-like vaginal odor B) Increased abdominal girth C) Fever and chills D) Lower abdominal pelvic pain

B) Increased abdominal girth

A firefighter was trapped in a fire and is admitted to the ICU for smoke inhalation. After 12 hours, the firefighter is exhibiting signs of ARDS and is intubated. What other supportive measures are initiated in a patient with ARDS? A) Psychological counseling B) Nutritional support C) High-protein oral diet D) Occupational therapy

B) Nutritional support

A 55-year-old man presents at the clinic complaining of erectile dysfunction. The patient has a history of diabetes. The physician orders tadalafil (Cialis) to be taken 1 hour before sexual intercourse. The nurse reviews the patients history prior to instructing the patient on the use of this medication. What disorder will contraindicate the use of tadalafil (Cialis)? A) Cataracts B) Retinopathy C) Hypotension D) Diabetic nephropathy

B) Retinopathy

The nurse is caring for a patient who has just had a radical mastectomy and axillary node dissection. When providing patient education regarding rehabilitation, what should the nurse recommend? A) Avoid exercise of the arm for next 2 months. B) Keep cuticles clipped neatly. C) Avoid lifting objects heavier than 10 pounds. D) Use a sling until healing is complete.

C) Avoid lifting objects heavier than 10 pounds.

A clinic nurse is providing preprocedure education for a man who will undergo a vasectomy. Which of the following measures will enhance healing and comfort? Select all that apply. A) Abstaining from sexual intercourse for at least 14 days postprocedure B) Wearing a scrotal support garment C) Using sitz baths D) Applying a heating pad intermittently E) Staying on bed rest for 48 to 72 hours postprocedure

B) Wearing a scrotal support garment C) Using sitz baths

A 42 year-old patient tells the nurse that she has found a painless lump in her right breast during her monthly self-examination. She says that she is afraid that she has cancer. Which assessment finding would most strongly suggest that this patients lump is cancerous? A) Eversion of the right nipple and mobile mass B) A nonmobile mass with irregular edges C) A mobile mass that is soft and easily delineated D) Nonpalpable right axillary lymph nodes

B) A nonmobile mass with irregular edges

The nurse is creating the plan of care for a patient who is status postsurgery for reduction of a femur fracture. What is the most important short-term goal for this patient? A) Relief of pain B) Adequate respiratory function C) Resumption of activities of daily living (ADLs) D) Unimpaired wound healing

B) Adequate respiratory function

A patient who is scheduled for an open prostatectomy is concerned about the potential effects of the surgery on his sexual function. What aspect of prostate surgery should inform the nurses response? A) Erectile dysfunction is common after prostatectomy as a result of hormonal changes. B) All prostatectomies carry a risk of nerve damage and consequent erectile dysfunction. C) Erectile dysfunction after prostatectomy is expected, but normally resolves within several months. D) Modern surgical techniques have eliminated the risk of erectile dysfunction following prostatectomy.

B) All prostatectomies carry a risk of nerve damage and consequent erectile dysfunction.

A public health nurse is teaching a mother about vaccinations prior to obtaining informed consent for her childs vaccination. What should the nurse cite as the most common adverse effect of vaccinations? A) Temporary sensitivity to the sun B) Allergic reactions to the antigen or carrier solution C) Nausea and vomiting D) Joint pain near the injection site

B) Allergic reactions to the antigen or carrier solution

A patient at high risk for breast cancer is scheduled for an incisional biopsy in the outpatient surgery department. When the nurse is providing preoperative education, the patient asks why an incisional biopsy is being done instead of just removing the mass. What would be the nurses best response? A) An incisional biopsy is performed because its known to be less painful and more accurate than other forms of testing. B) An incisional biopsy is performed to confirm a diagnosis and so that special studies can be done that will help determine the best treatment. C) An incisional biopsy is performed to assess the potential for recovery from a mastectomy. D) An incisional biopsy is performed on patients who are younger than the age of 40 and who are otherwise healthy.

B) An incisional biopsy is performed to confirm a diagnosis and so that special studies can be done that will help determine the best treatment.

A patient is post-operative day 1 following a vaginal hysterectomy. The nurse notes an increase in the patients abdominal girth and the patient complains of bloating. What is the nurses most appropriate action? A) Provide the patient with an unsweetened, carbonated beverage. B) Apply warm compresses to the patients lower abdomen. C) Provide an ice pack to apply to the perineum and suprapubic region. D) Assist the patient into a prone position.

B) Apply warm compresses to the patients lower abdomen.

A 31-year-old patient has returned to the post-surgical unit following a hysterectomy. The patients care plan addresses the risk of hemorrhage. How should the nurse best monitor the patients postoperative blood loss? A) Have the patient void and have bowel movements using a commode rather than toilet. B) Count and inspect each perineal pad that the patient uses. C) Swab the patients perineum for the presence of blood at least once per shift. D) Leave the patients perineum open to air to facilitate inspection.

B) Count and inspect each perineal pad that the patient uses.

A patient is admitted from the ED diagnosed with Neisseria meningitides. What type of isolation precautions should the nurse institute? A) Contact precautions B) Droplet precautions C) Airborne precautions D) Observation precautions

B) Droplet precautions

The nurse is preparing to change a patients abdominal dressing. The nurse recognizes the first step is to provide the patient with information regarding the procedure. Which of the following explanations should the nurse provide to the patient? A) The dressing change is often painful, and we will be giving you pain medication prior to the procedure so you do not have to worry. B) During the dressing change, I will provide privacy at a time of your choosing, it should not be painful, and you can look at the incision and help with the procedure if you want to. C) The dressing change should not be painful, but you can never be sure, and infection is always a concern. D) The best time for doing a dressing change is during lunch so we are not interrupted. I will provide privacy, and it should not be painful.

B) During the dressing change, I will provide privacy at a time of your choosing, it should not be painful, and you can look at the incision and help with the procedure if you want to.

The nurse is caring for a patient with chronic obstructive pulmonary disease (COPD). The patient has been receiving high-flow oxygen therapy for an extended time. What symptoms should the nurse anticipate if the patient were experiencing oxygen toxicity? A) Bradycardia and frontal headache B) Dyspnea and substernal pain C) Peripheral cyanosis and restlessness D) Hypotension and tachycardia

B) Dyspnea and substernal pain

The nurse is being trained to perform assessment screenings for abuse on patients who come into the walk-in clinic where the nurse works. Which of the following assessment questions is most appropriate? A) Would you describe your relationship as healthy and functional? B) Have you ever been forced into sexual activity? C) Do you make your husband uncontrollably angry? D) How is conflict usually handled in your home?

B) Have you ever been forced into sexual activity?

A newly pregnant patient is being assessed in an obstetric clinic. The patient states that she has been experiencing intense abdominal pain and the nurse anticipates that the patient will be assessed for ectopic pregnancy. In addition to ultrasonography, what diagnostic test should the nurse anticipate? A) Computed tomography B) Human chorionic gonadotropin (hCG) testing C) Estrogen and progesterone testing D) Abdominal x-ray

B) Human chorionic gonadotropin (hCG) testing

When teaching patients about the risk factors of cervical cancer, what would the nurse identify as the most important risk factor? A) Late childbearing B) Human papillomavirus (HPV) C) Postmenopausal bleeding D) Tobacco use

B) Human papillomavirus (HPV)

A patient has returned to the floor from the PACU after undergoing a suprapubic prostatectomy. The nurse notes significant urine leakage around the suprapubic tube. What is the nurses most appropriate action? A) Cleanse the skin surrounding the suprapubic tube. B) Inform the urologist of this finding. C) Remove the suprapubic tube and apply a wet-to-dry dressing. D) Administer antispasmodic drugs as ordered.

B) Inform the urologist of this finding.

A 42-year-old man has come to the clinic for an annual physical. The nurse notes in the patients history that his father was treated for breast cancer. What should the nurse provide to the patient before he leaves the clinic? A) A referral for a mammogram B) Instructions about breast self-examination (BSE) C) A referral to a surgeon D) A referral to a support group

B) Instructions about breast self-examination (BSE)

The nurse is caring for a patient with lung metastases who just underwent a mediastinotomy. What should be the focus of the nurses postprocedure care? A) Assisting with pulmonary function testing (PFT) B) Maintaining the patients chest tube C) Administering oral suction as needed D) Performing chest physiotherapy

B) Maintaining the patients chest tube

A 45-year-old woman comes into the health clinic for her annual check-up. She mentions to the nurse that she has noticed dimpling of the right breast that has occurred in a few months. What assessment would be most appropriate for the nurse to make? A) Evaluate the patients milk production. B) Palpate the area for a breast mass. C) Assess the patients knowledge of breast cancer. D) Assure the patient that this likely an age-related change.

B) Palpate the area for a breast mass.

The nursing instructor is discussing the difference between ambulatory surgical centers and hospital- based surgical units. A student asks why some patients have surgery in the hospital and others are sent to ambulatory surgery centers. What is the instructors best response? A) Patients who go to ambulatory surgery centers are more independent than patients admitted to the hospital. B) Patients admitted to the hospital for surgery usually have multiple health needs. C) In most cases, only emergency and trauma patients are admitted to the hospital. D) Patients who have surgery in the hospital are those who need to have anesthesia administered.

B) Patients admitted to the hospital for surgery usually have multiple health needs.

The nurse is caring for a patient with a diagnosis of vulvar cancer who has returned from the PACU after undergoing a wide excision of the vulva. How should this patients analgesic regimen be best managed? A) Analgesia should be withheld unless the patients pain becomes unbearable. B) Scheduled analgesia should be administered around-the-clock to prevent pain. C) All analgesics should be given on a PRN, rather than scheduled, basis. D) Opioid analgesics should be avoided and NSAIDs exclusively provided.

B) Scheduled analgesia should be administered around-the-clock to prevent pain.

A surgical patient has been in the PACU for the past 3 hours. What are the determining factors for the patient to be discharged from the PACU? Select all that apply. A) Absence of pain B) Stable blood pressure C) Ability to tolerate oral fluids D) Sufficient oxygen saturation E) Adequate respiratory function

B) Stable blood pressure D) Sufficient oxygen saturation E) Adequate respiratory function

A 75-year-old male patient is being treated for phimosis. When planning this patients care, what health promotion activity is most directly related to the etiology of the patients health problem? A) Teaching the patient about safer sexual practices B) Teaching the patient about the importance of hygiene C) Teaching the patient about the safe use of PDE-5 inhibitors D) Teaching the patient to perform testicular self-examination

B) Teaching the patient about the importance of hygiene

A new patient has come to the clinic seeking an appropriate method of birth control. What would the nurse teach this patient about a diaphragm? A) One size fits all females. B) The diaphragm may be cleaned with soap and water after use. C) A diaphragm eliminates the need for spermicidal jelly. D) The diaphragm should be removed 1 hour following intercourse. Ans: B

B) The diaphragm may be cleaned with soap and water after use.

A patient has been prescribed sildenafil. What should the nurse teach the patient about this medication? A) Sexual stimulation is not needed to obtain an erection. B) The drug should be taken 1 hour prior to intercourse. C) Facial flushing or headache should be reported to the physician immediately. D) The drug has the potential to cause permanent visual changes.

B) The drug should be taken 1 hour prior to intercourse.

The nurse is performing a comprehensive health history of a patient who is in her 50s. The nurse should identify what risk factor that may increase this patients risk for breast cancer? A) The patient breastfed each of her children. B) The patient gave birth to her first child at age 38. C) The patient experienced perimenopausal symptoms starting at age 46. D) The patient experienced menarche at age 13.

B) The patient gave birth to her first child at age 38.

A nurse is examining a patient who has been diagnosed with a fibroadenoma. The nurse should recognize what implication of this patients diagnosis? A) The patient will be scheduled for radiation therapy. B) The patient might be referred for a biopsy. C) The patients breast mass is considered an age-related change. D) The patients diagnosis is likely related to her use of oral contraceptives.

B) The patient might be referred for a biopsy.

A patient diagnosed with a pituitary adenoma has arrived on the neurologic unit. When planning the patients care, the nurse should be aware that the effects of the tumor will primarily depend on what variable? A) Whether the tumor utilizes aerobic or anaerobic respiration B) The specific hormones secreted by the tumor C) The patients pre-existing health status D) Whether the tumor is primary or the result of metastasis

B) The specific hormones secreted by the tumor

A 48-year-old woman presenting for care is seeking information about hormone therapy (HT) for the treatment of her perimenopausal symptoms. The patients need for relief from hot flashes and other symptoms will be weighed carefully against the increased risks of what complications of HT? Select all that apply. A) Anaphylaxis B) Osteoporosis C) Breast cancer D) Cardiovascular disease E) Venous thromboembolism

C) Breast cancer D) Cardiovascular disease E) Venous thromboembolism

While taking a health history on a 20-year-old female patient, the nurse ascertains that this patient is taking miconazole (Monistat). The nurse is justified in presuming that this patient has what medical condition? A) Bacterial vaginosis B) Human papillomavirus (HPV) C) Candidiasis D) Toxic shock syndrome (TSS)

C) Candidiasis

A patient presents to the ED stating she was in a boating accident about 3 hours ago. Now the patient has complaints of headache, fatigue, and the feeling that he just cant breathe enough. The nurse notes that the patient is restless and tachycardic with an elevated blood pressure. This patient may be in the early stages of what respiratory problem? A) Pneumoconiosis B) Pleural effusion C) Acute respiratory failure D) Pneumonia

C) Acute respiratory failure

A woman aged 48 years comes to the clinic because she has discovered a lump in her breast. After diagnostic testing, the woman receives a diagnosis of breast cancer. The woman asks the nurse when her teenage daughters should begin mammography. What is the nurses best advice? A) Age 28 B) Age 35 C) Age 38 D) Age 48

C) Age 38

A student nurse is caring for a patient who has undergone a wide excision of the vulva. The student should know that what action is contraindicated in the immediate postoperative period? A) Placing patient in low Fowlers position B) Application of compression stockings C) Ambulation to a chair D) Provision of a low-residue diet

C) Ambulation to a chair

The nurse just received a postoperative patient from the PACU to the medicalsurgical unit. The patient is an 84-year-old woman who had surgery for a left hip replacement. Which of the following concerns should the nurse prioritize for this patient in the first few hours on the unit? A) Beginning early ambulation B) Maintaining clean dressings on the surgical site C) Close monitoring of neurologic status D) Resumption of normal oral intake

C) Close monitoring of neurologic status

A patients diagnostic testing revealed that he is colonized with vancomycin-resistantenterococcus (VRE). What change in the patients health status could precipitate an infection? A) Use of a narrow-spectrum antibiotic B) Treatment of a concurrent infection using vancomycin C) Development of a skin break D) Persistent contact of the bacteria with skin surfaces

C) Development of a skin break

The nurse is planning the care of a patient who is scheduled for a laryngectomy. The nurse should assign the highest priority to which postoperative nursing diagnosis? A) Anxiety related to diagnosis of cancer B) Altered nutrition related to swallowing difficulties C) Ineffective airway clearance related to airway alterations D) Impaired verbal communication related to removal of the larynx

C) Ineffective airway clearance related to airway alterations

A nurse practitioner is preparing to perform a patients scheduled Pap smear and the patient asks the nurse to ensure that the speculum is well-lubricated. How should the nurse proceed with assessment? A) Reassure the patient that ample petroleum jelly will be used. B) Reassure that patient that a water-based lubricant will be used. C) Explain to the patient that water is the only lubricant that can be used. D) Explain to the patient why the speculum must be introduced dry.

C) Explain to the patient that water is the only lubricant that can be used.

A 23-year-old woman comes to the free clinic stating I think I have a lump in my breast. Do I have cancer? The nurse instructs the patient that a diagnosis of breast cancer is confirmed by what? A) Supervised breast self-examination B) Mammography C) Fine-needle aspiration D) Chest x-ray

C) Fine-needle aspiration

The nurse is providing care for an older adult patient who has developed signs and symptoms ofCalicivirus (Norovirus). What assessment should the nurse prioritize when planning this patients care? A) Respiratory status B) Pain C) Fluid intake and output D) Deep tendon reflexes and neurological status

C) Fluid intake and output

A couple with a diagnosis of ovarian failure discusses their infertility options with their physician. The nurse should recognize which of the following as the treatment of choice for a patient with ovarian failure? A) Intracytoplasmic sperm injection B) Artificial insemination C) Gamete intrafallopian transfer D) In vitro fertilization

C) Gamete intrafallopian transfer

A patient in her late fifties has expressed to the nurse her desire to explore hormone replacement therapy (HRT). Based on what aspect of the patients health history is HRT contraindicated? A) History of vaginal dryness B) History of hot flashes and night sweats C) History of vascular thrombosis D) Family history of osteoporosis

C) History of vascular thrombosis

An adolescent is identified as having a collection of fluid in the tunica vaginalis of his testes. The nurse knows that this adolescent will receive what medical diagnosis? A) Cryptorchidism B) Orchitis C) Hydrocele D) Prostatism

C) Hydrocele

You are caring for a 71-year-old patient who is 4 days postoperative for bilateral inguinal hernias. The patient has a history of congestive heart failure and peptic ulcer disease. The patient is highly reluctant to ambulate and will not drink fluids except for hot tea with her meals. The nurses aide reports to you that this patients vital signs are slightly elevated and that she has a nonproductive cough. When you assess the patient, you auscultate crackles at the base of the lungs. What would you suspect is wrong with your patient? A) Pulmonary embolism B) Hypervolemia C) Hypostatic pulmonary congestion D) Malignant hyperthermia

C) Hypostatic pulmonary congestion

The nurse is caring for a patient in the ICU admitted with ARDS after exposure to toxic fumes from a hazardous spill at work. The patient has become hypotensive. What is the cause of this complication to the ARDS treatment? A) Pulmonary hypotension due to decreased cardiac output B) Severe and progressive pulmonary hypertension C) Hypovolemia secondary to leakage of fluid into the interstitial spaces D) Increased cardiac output from high levels of PEEP therapy

C) Hypovolemia secondary to leakage of fluid into the interstitial spaces

A patient has returned to the post-surgical unit after vulvar surgery. What intervention should the nurse prioritize during the initial postoperative period? A) Placing the patient in high Fowlers position B) Administering sitz baths every 4 hours C) Monitoring the integrity of the surgical site D) Avoiding analgesics unless the patients pain is unbearable

C) Monitoring the integrity of the surgical site

A 30-year-old patient has come to the clinic for her yearly examination. The patient asks the nurse about ovarian cancer. What should the nurse state when describing risk factors for ovarian cancer? A) Use of oral contraceptives increases the risk of ovarian cancer. B) Most cases of ovarian cancer are attributed to tobacco use. C) Most cases of ovarian cancer are considered to be random, with no obvious causation. D) The majority of women who get ovarian cancer have a family history of the disease.

C) Most cases of ovarian cancer are considered to be random, with no obvious causation.

A gerontologic nurse is teaching a group of medical nurses about the high incidence and mortality of pneumonia in older adults. What is a contributing factor to this that the nurse should describe? A) Older adults have less compliant lung tissue than younger adults. B) Older adults are not normally candidates for pneumococcal vaccination. C) Older adults often lack the classic signs and symptoms of pneumonia. D) Older adults often cannot tolerate the most common antibiotics used to treat pneumonia.

C) Older adults often lack the classic signs and symptoms of pneumonia.

A patient asks the nurse why an infection in his upper respiratory system is affecting the clarity of his speech. Which structure serves as the patients resonating chamber in speech? A) Trachea B) Pharynx C) Paranasal sinuses D) Larynx

C) Paranasal sinuses

A nurse is assessing a patient who presented to the ED with priapism. The student nurse is aware that this condition is classified as a urologic emergency because of the potential for what? A) Urinary tract infection B) Chronic pain C) Permanent vascular damage D) Future erectile dysfunction

C) Permanent vascular damage

An uncircumcised 78-year-old male has presented at the clinic complaining that he cannot retract his foreskin over his glans. On examination, it is noted that the foreskin is very constricted. The nurse should recognize the presence of what health problem? A) Bowens disease B) Peyronies disease C) Phimosis D) Priapism

C) Phimosis

The nurse is caring for a 78-year-old man who has had an outpatient cholecystectomy. The nurse is getting him up for his first walk postoperatively. To decrease the potential for orthostatic hypotension and consequent falls, what should the nurse have the patient do? A) Sit in a chair for 10 minutes prior to ambulating. B) Drink plenty of fluids to increase circulating blood volume. C) Stand upright for 2 to 3 minutes prior to ambulating. D) Perform range-of-motion exercises for each joint.

C) Stand upright for 2 to 3 minutes prior to ambulating.

A woman scheduled for a simple mastectomy in one week is having her preoperative education provided by the clinic nurse. What educational intervention will be of primary importance to prevent hemorrhage in the postoperative period? A) Limit her intake of green leafy vegetables. B) Increase her water intake to 8 glasses per day. C) Stop taking aspirin. D) Have nothing by mouth for 6 hours before surgery.

C) Stop taking aspirin.

When planning discharge teaching with a patient who has undergone a total mastectomy with axillary dissection, the nurse knows to instruct the patient that she should report what sign or symptom to the physician immediately? A) Fatigue B) Temperature greater than 98.5F C) Sudden cessation of output from the drainage device D) Gradual decline in output from the drain

C) Sudden cessation of output from the drainage device

The nurse is caring for a 46-year-old patient recently diagnosed with the early stages of lung cancer. The nurse is aware that the preferred method of treating patients with nonsmall cell tumors is what? A) Chemotherapy B) Radiation C) Surgical resection D) Bronchoscopic opening of the airway

C) Surgical resection

A nursing student is discussing a patient with viral pharyngitis with the preceptor at the walk-in clinic. What should the preceptor tell the student about nursing care for patients with viral pharyngitis? A) Teaching focuses on safe and effective use of antibiotics. B) The patient should be preliminarily screened for surgery. C) Symptom management is the main focus of medical and nursing care. D) The focus of care is resting the voice to prevent chronic hoarseness.

C) Symptom management is the main focus of medical and nursing care.

The school nurse is presenting a class on female reproductive health. The nurse should describe what aspect of Pap smears? A) The test may be performed at any time during the patients menstrual cycle. B) The smear should be done every 2 years. C) The test can detect early evidence of cervical cancer. D) Falsepositive Pap smear results occur mostly from not douching before the examination.

C) The test can detect early evidence of cervical cancer.

A patient has just been diagnosed with lung cancer. After the physician discusses treatment options and leaves the room, the patient asks the nurse how the treatment is decided upon. What would be the nurses best response? A) The type of treatment depends on the patients age and health status. B) The type of treatment depends on what the patient wants when given the options. C) The type of treatment depends on the cell type of the cancer, the stage of the cancer, and the patients health status. D) The type of treatment depends on the discussion between the patient and the physician of which treatment is best.

C) The type of treatment depends on the cell type of the cancer, the stage of the cancer, and the patients health status.

A client presents to the walk-in clinic complaining of a dry, irritating cough and production of a minute amount of mucus-like sputum. The patient complains of soreness in her chest in the sternal area. The nurse should suspect that the primary care provider will assess the patient for what health problem? A) Pleural effusion B) Pulmonary embolism C) Tracheobronchitis D) Tuberculosis

C) Tracheobronchitis

A patient is being discharged home after a hysterectomy. When providing discharge education for this patient, the nurse has cautioned the patient against sitting for long periods. This advice addresses the patients risk of what surgical complication? A) Pudendal nerve damage B) Fatigue C) Venous thromboembolism D) Hemorrhage

C) Venous thromboembolism

The nurse is taking the sexual history of an adolescent who has come into the free clinic. What question best assesses the patients need for further information? A) Are you involved in an intimate relationship at this time? B) How many sexual partners have you had? C) What questions or concerns do you have about your sexual health? D) Have you ever been diagnosed with a sexually transmitted infection?

C) What questions or concerns do you have about your sexual health?

For which of the following population groups would an annual clinical breast examination be recommended? A) Women over age 21 B) Women over age 25 C) Women over age 40 D) All post-pubescent females with a family history of breast cancer

C) Women over age 40

5. The nurse is caring for a patient admitted with an acute exacerbation of chronic obstructive pulmonary disease. During assessment, the nurse finds that the patient is experiencing increased dyspnea. What is the most accurate measurement of the concentration of oxygen in the patients blood? A) A capillary blood sample B) Pulse oximetry C)An arterial blood gas (ABG) study d)A complete blood count (CBC)

C)An arterial blood gas (ABG) study

A patient in her 30s has two young children and has just had a modified radical mastectomy with immediate reconstruction. The patient shares with the nurse that she is somewhat worried about her future, but she appears to be adjusting well to her diagnosis and surgery. What nursing intervention is most appropriate to support this patients coping? A) Encourage the patients spouse or partner to be supportive while she recovers. B) Encourage the patient to proceed with the next phase of treatment. C) Recommend that the patient remain optimistic for the sake of her children. D) Arrange a referral to a community-based support program.

D) Arrange a referral to a community-based support program

The nurse is teaching a patient preventative measures regarding vaginal infections. The nurse should include which of the following as an important risk factor? A) High estrogen levels B) Late menarche C) Nonpregnant state D) Frequent douching

D) Frequent douching

A patient states that PMS that is significantly disrupting her quality of life and that conservative management has failed to produce relief. What pharmacologic treatment may benefit this patient? A) An opioid analgesic B) A calcium channel blocker C) A monoamine oxidase inhibitor (MAOI) D) A selective serotonin reuptake inhibitor (SSRI)

D) A selective serotonin reuptake inhibitor (SSRI)

A 35-year-old mother of three young children has been diagnosed with stage II breast cancer. After discussing treatment options with her physician, the woman goes home to talk to her husband, later calling the nurse for clarification of some points. The patient tells the nurse that the physician has recommended breast conservation surgery followed by radiation. The patients husband has done some online research and is asking why his wife does not have a modified radical mastectomy to be sure all the cancer is gone. What would be the nurses best response? a. Modified radical mastectomies are very hard on a patient, both physically and emotionally and they really arent necessary anymore. b. According to current guidelines, having a modified radical mastectomy is no longer seen as beneficial. C) Modified radical mastectomies have a poor survival rate because of the risk of cancer recurrence. D) According to current guidelines, breast conservation combined with radiation is as effective as a modified radical mastectomy.

D) According to current guidelines, breast conservation combined with radiation is as effective as a modified radical mastectomy.

A patient has returned to the floor after undergoing a transurethral resection of the prostate (TURP). The patient has a continuous bladder irrigation system in place. The patient tells you he is experiencing bladder spasms and asks what you can do to relieve his discomfort. What is the most appropriate nursing action to relieve the discomfort of the patient? A) Apply a cold compress to the pubic area. B) Notify the urologist promptly. C) Irrigate the catheter with 30 to 50 mL of normal saline as ordered. D) Administer a smooth-muscle relaxant as ordered.

D) Administer a smooth-muscle relaxant as ordered.

The recovery room nurse is admitting a patient from the OR following the patients successful splenectomy. What is the first assessment that the nurse should perform on this newly admitted patient? A) Heart rate and rhythm B) Skin integrity C) Core body temperature D) Airway patency

D) Airway patency

The nurse is teaching breast self-examination (BSE) to a group of women. The nurse should recommend that the women perform BSE at what time? A) At the time of menses B) At any convenient time, regardless of cycles C) Weekly D) Between days 5 and 7 after menses

D) Between days 5 and 7 after menses

A woman is being treated for a tumor of the left breast. If the patient and her physician opt for prophylactic treatment, the nurse should prepare the woman for what intervention? A) More aggressive chemotherapy B) Left mastectomy C) Radiation therapy D) Bilateral mastectomy

D) Bilateral mastectomy

A community health nurse is leading a health education session addressing menopause and other aspects of womens health. What dietary supplements should the nurse recommend to prevent morbidity associated with osteoporotic fractures? A) Vitamin B12 and vitamin C B) Vitamin A and potassium C) Vitamin B6 and phosphorus D) Calcium and vitamin D

D) Calcium and vitamin D

An 87-year-old patient has been hospitalized with pneumonia. Which nursing action would be a priority in this patients plan of care? A) Nasogastric intubation B) Administration of probiotic supplements C) Bedrest D) Cautious hydration

D) Cautious hydration

A patient is 24 hours postoperative following prostatectomy and the urologist has ordered continuous bladder irrigation. What color of output should the nurse expect to find in the drainage bag? A) Red wine colored B) Tea colored C) Amber D) Light pink

D) Light pink

A nurse is planning the postoperative care of a patient who is scheduled for radical prostatectomy. What intraoperative position will place the patient at particular risk for the development of deep vein thrombosis postoperatively? A) Fowlers position B) Prone position C) Supine position D) Lithotomy position

D) Lithotomy position

A patient confides to the nurse that he cannot engage in sexual activity. The patient is 27 years old and has no apparent history of chronic illness that would contribute to erectile dysfunction. What does the nurse know will be ordered for this patient to assess his sexual functioning? A) Sperm count B) Ejaculation capacity tests C) Engorgement tests D) Nocturnal penile tumescence tests

D) Nocturnal penile tumescence tests

A patient newly diagnosed with breast cancer states that her physician suspects regional lymph node involvement and told her that there are signs of metastatic disease. The nurse learns that the patient has been diagnosed with stage IV breast cancer. What is an implication of this diagnosis? A) The patient is not a surgical candidate. B) The patients breast cancer is considered highly treatable. C) There is a 10% chance that the patients cancer will self-resolve. D) The patient has a 15% chance of 5-year survival.

D) The patient has a 15% chance of 5-year survival.

A student nurse is doing clinical hours at an OB/GYN clinic. The student is helping to develop a plan of care for a patient with gonorrhea who has presented at the clinic. The student should include which of the following in the care plan for this patient? A) The patient may benefit from oral contraceptives. B) The patient must avoid use of tampons. C) The patient is susceptible to urinary incontinence. D) The patient should also be treated for chlamydia.

D) The patient should also be treated for chlamydia.

A patient has just been diagnosed with prostate cancer and is scheduled for brachytherapy next week. The patient and his wife are unsure of having the procedure because their daughter is 3 months pregnant. What is the most appropriate teaching the nurse should provide to this family? A) The patient should not be in contact with the baby after delivery. B) The patients treatment poses no risk to his daughter or her infant. C) The patients brachytherapy may be contraindicated for safety reasons. D) The patient should avoid close contact with his daughter for 2 months.

D) The patient should avoid close contact with his daughter for 2 months.

The nurse is caring for a patient who has been in a motor vehicle accident and the care team suspects that the patient has developed pleurisy. Which of the nurses assessment findings would best corroborate this diagnosis? A) The patient is experiencing painless hemoptysis. B) The patients arterial blood gases (ABGs) are normal, but he demonstrates increased work of breathing. C) The patients oxygen saturation level is below 88%, but he denies shortness of breath. D) The patients pain intensifies when he coughs or takes a deep breath.

D) The patients pain intensifies when he coughs or takes a deep breath.

A patient has herpes simplex 2 viral infection (HSV2). The nurse recognizes that which of the following should be included in teaching the patient? A) The virus causes cold sores of the lips. B) The virus may be cured with antibiotics. C) The virus, when active, may not be contracted during intercourse. D) Treatment is aimed at relieving symptoms.

D) Treatment is aimed at relieving symptoms.

An adult patient is in the recovery room following a nephrectomy performed for the treatment of renal cell carcinoma. The patients vital signs and level of consciousness stabilized, but the patient then complains of severe nausea and begins to retch. What should the nurse do next? A) Administer a dose of IV analgesic. B) Apply a cool cloth to the patients forehead. C) Offer the patient a small amount of ice chips. D) Turn the patient completely to one side.

D) Turn the patient completely to one side.

A 51-year-old woman has come to the OB/GYN clinic for her annual physical. She tells the nurse that she has been experiencing severe hot flashes, but that she is reluctant to begin hormone therapy (HT). What potential solution should the nurse discuss with the patient? A) Sodium restriction B) Adopting a vegan diet C) Massage therapy D) Vitamin supplements

D) Vitamin supplements

5. A female patient who has cognitive and physical disabilities has come into the clinic for a routine check- up. When planning this patients assessment, what action should the nurse take? A) Ensure that a chaperone is available to be present during the assessment. B)Limit the length and scope of the health assessment. C)Avoid health promotion or disease prevention education. D)Avoid equating the patient with her disabilities.

D)Avoid equating the patient with her disabilities.

A student nurse is learning about the health care needs of lesbian, gay, bisexual, transgender, and queer/questioning (LGBTQ) clients. Which terms are correctly defined? (Select all that apply.) a. Gender dysphoria Distress caused by incongruence between natal sex and gender identity b. Gender queer A label used when gender identity does not conform to male or female c. Natal sex The sex one is born with or is assigned to at birth d. Transgender A person who dresses in the clothing of the opposite sex e. Transition The time between questioning and establishing a sexual identity

a. Gender dysphoria Distress caused by incongruence between natal sex and gender identity b. Gender queer A label used when gender identity does not conform to male or female c. Natal sex The sex one is born with or is assigned to at birth

A nurse works with many transgender clients. What routine monitoring is important for the nurse to facilitate in this population? (Select all that apply.) a. Lipid profile b. Liver function tests c. Mammograms if breast tissue is present d. Prostate-specific antigen (PSA) for natal males e. Renal profile

a. Lipid profile b. Liver function tests c. Mammograms if breast tissue is present d. Prostate-specific antigen (PSA) for natal males

To decrease glandular cellular activity and prostate size, an 83-year-old patient has been prescribed finasteride (Proscar). When performing patient education with this patient, the nurse should be sure to tell the patient what? a. Report the planned use of dietary supplements to the physician. b. Decrease the intake of fluids to prevent urinary retention. c. Abstain from sexual activity for 2 weeks following the initiation of treatment. d. Anticipate a temporary worsening of urinary retention before symptoms subside.

a. Report the planned use of dietary supplements to the physician.

A gerontologic nurse is analyzing the data from a patients focused respiratory assessment. The nurse is aware that the amount of respiratory dead space increases with age. What is the effect of this physiological change? a)Increased diffusion of gases b)Decreased diffusion capacity for oxygen c)Decreased shunting of blood d)Increased ventilation

b)Decreased diffusion capacity for oxygen

A nurse presenting an educational event for a local community group is addressing premenstrual syndrome (PMS). What treatment guideline should the nurse teach this group? a.Avoid excessive fluid intake. b.Increase the frequency and intensity of exercise. c.Limit psychosocial stressors in order to reduce symptoms. d.Take opioid analgesics as ordered.

b.Increase the frequency and intensity of exercise.

The nurse is utilizing the PLISSIT model of sexual health assessment during an interaction with a new patient. According to this model, the nurse should begin with what action? A) Conducting a preliminary assessment b. Addressing the patients psychosocial status c. Asking the patients permission to discuss sexuality d. Assessing for physiologic problems

c. Asking the patients permission to discuss sexuality

A nurse is providing an educational event to a local mens group about prostate cancer. The nurse should cite an increased risk of prostate cancer in what ethnic group? a. Native Americans b. Caucasian Americans c. African Americans d. Asian Americans

c. African Americans

When reviewing the electronic health record of a female patient, the nurse reads that the patient has a history of adenomyosis. The nurse should be aware that this patient experiences symptoms resulting from what pathophysiologic process? A) Loss of muscle tone in the vaginal wall b. Excessive synthesis and release of unopposed estrogen c. Invasion of the uterine wall by endometrial tissue d. Proliferation of tumors in the uterine wall

c. Invasion of the uterine wall by endometrial tissue

A patient has just been diagnosed with breast cancer and the nurse is performing a patient interview. In assessing this patients ability to cope with this diagnosis, what would be an appropriate question for the nurse to ask this patient? a. What is your level of education? b. Are you feeling alright these days? c. Is there someone you trust to help you make treatment choices? d. Are you concerned about receiving this diagnosis?

c. Is there someone you trust to help you make treatment choices?

A transgender client taking spironolactone (Aldactone) is in the internal medicine clinic reporting heart palpitations. What action by the nurse takes priority? a. Draw blood to test serum potassium. b. Have the client lie down. c. Obtain a STAT electrocardiogram (ECG). d. Take a set of vital signs.

c. Obtain a STAT electrocardiogram (ECG).

During a recent visit to the clinic a woman presents with erythema of the nipple and areola on the right breast. She states this started several weeks ago and she was fearful of what would be found. The nurse should promptly refer the patient to her primary care provider because the patients signs and symptoms are suggestive of what health problem? A) Peau dorange B) Nipple inversion c. Pagets disease d. Acute mastitis

c. Pagets disease

The nurse is caring for a 79-year-old man who has returned to the postsurgical unit following abdominal surgery. The patient is unable to ambulate and is now refusing to wear an external pneumatic compression stocking. The nurse should explain that refusing to wear external pneumatic compression stockings increases his risk of what postsurgical complication? A) Sepsis b. Infection c. Pulmonary embolism d. Hematoma

c. Pulmonary embolism

A transgender client is taking transdermal estrogen (Climara). What assessment finding does the nurse report immediately to the provider? a. Breast tenderness b. Headaches c. Red, swollen calf d. Swollen ankles

c. Red, swollen calf

The nurse is assessing a patient who believes that she has recently begun menopause. What principle should inform the nurses interactions with this patient? a. The nurse should express empathy for the patients difficult health situation. b. The nurse should begin by assuring the patient that her health will be much better in a few years. c. The nurse must carefully assess the patients feelings and beliefs surrounding menopause. d. The nurse should encourage the patient to celebrate this life milestone and its accompanying benefits.

c. The nurse must carefully assess the patients feelings and beliefs surrounding menopause.

A patient with genital herpes is having an acute exacerbation. What medication would the nurse expect ordered to suppress the symptoms and shorten the course of the infection? a. Clotrimazole (Gyne-Lotrimin) b. Metronidazole (Flagyl) c. Podophyllin (Podofin) d. Acyclovir (Zovirax)

d. Acyclovir (Zovirax)

The nurse is providing teaching about tissue repair and wound healing to a patient who has a leg ulcer. Which of the following statements by the patient indicates that teaching has been effective? a. Ill make sure to limit my intake of protein. b. Ill make sure that the bandage is wrapped tightly. c. My foot should feel cool or cold while my legs healing. d. Ill eat plenty of fruits and vegetables.

d. Ill eat plenty of fruits and vegetables.

A woman is considering breast reduction mammoplasty. When weighing the potential risks and benefits of this surgical procedure, the nurse should confirm that the patient is aware of what potential consequence? a. Chronic breast pain b. Unclear mammography results c. Increased risk of breast cancer d. Decreased nipple sensation

d. Decreased nipple sensation

The nursing instructor is talking with a group of medicalsurgical students about deep vein thrombosis (DVT). A student asks what factors contribute to the formation of a DVT. What would be the instructors best response? a. There is a genetic link in the formation of deep vein thrombi. b. Hypervolemia is often present in patients who go on to develop deep vein thrombi. c. No known factors contribute to the formation of deep vein thrombi; they just occur. d. Dehydration is a contributory factor to the formation of deep vein thrombi.

d. Dehydration is a contributory factor to the formation of deep vein thrombi.

The nurse notes that a patient has a history of fibroids and is aware that this term refers to a benign tumor of the uterus. What is a more appropriate term for a fibroid? A) Bartholins cyst b. Dermoid cyst c. Hydatidiform mole d. Leiomyoma

d. Leiomyoma

The PACU nurse is caring for a male patient who had a hernia repair. The patients blood pressure is now 164/92 mm Hg; he has no history of hypertension prior to surgery and his preoperative blood pressure was 112/68 mm Hg. The nurse should assess for what potential causes of hypertension following surgery? a. Dysrhythmias, blood loss, and hyperthermia b. Electrolyte imbalances and neurologic changes c. A parasympathetic reaction and low blood volumes d. Pain, hypoxia, or bladder distention

d. Pain, hypoxia, or bladder distention

A patient has been diagnosed with erectile dysfunction; the cause has been determined to be psychogenic. The patients interdisciplinary plan of care should prioritize which of the following interventions? a. Penile implant b. PDE-5 inhibitors c. Physical therapy d. Psychotherapy

d. Psychotherapy

The medical nurse who works on a pulmonology unit is aware that several respiratory conditions can affect lung tissue compliance. The presence of what condition would lead to an increase in lung compliance? A) Emphysema B) Pulmonary fibrosis C) Pleural effusion D) Acute respiratory distress syndrome (ARDS)

A) Emphysema

A school nurse is presenting information on human development and sexuality. When describing the role of hormones in sexual development, which hormone does the nurse teach the class is the most important one for developing and maintaining the female reproductive organs? A) Estrogen B) Progesterone C) Androgens D) Follicle-stimulating hormone

A) Estrogen

The clinic nurse is caring for a patient who has been diagnosed with emphysema and who has just had a pulmonary function test (PFT) ordered. The patient asks, What exactly is this test for? What would be the nurses best response? a)A PFT measures how much air moves in and out of your lungs when you breathe. b)A PFT measures how much energy you get from the oxygen you breathe. c)A PFT measures how elastic your lungs are. d)A PFT measures whether oxygen and carbon dioxide move between your lungs and your blood.

A PFT measures how much air moves in and out of your lungs when you breathe.

While assessing a 25-year-old female, the nurse notes that the patient has hair on her lower abdomen. Earlier in the health interview, the patient stated that her menses are irregular. The nurse should suspect what type of health problem? a)A metabolic disorder b)A malignancy c)A hormonal imbalance D) An infectious process

A hormonal imbalance

A surgical patient has just been admitted to the unit from PACU with patient-controlled analgesia (PCA). The nurse should know that the requirements for safe and effective use of PCA include what? A) A clear understanding of the need to self-dose B) An understanding of how to adjust the medication dosage C) A caregiver who can administer the medication as ordered D) An expectation of infrequent need for analgesia

A) A clear understanding of the need to self-dose

A man tells the nurse that his father died of prostate cancer and he is concerned about his own risk of developing the disease, having heard that prostate cancer has a genetic link. What aspect of the pathophysiology of prostate cancer would underlie the nurses response? A) A number of studies have identified an association of BRCA-2 mutation with an increased risk of prostate cancer. B) HNPCC is a mutation of two genes that causes prostate cancer in men and it is autosomal dominant. C) Studies have shown that the presence of the TP53 gene strongly influences the incidence of prostate cancer. D) Recent research has demonstrated that prostate cancer is the result of lifestyle factors and that genetics are unrelated.

A) A number of studies have identified an association of BRCA-2 mutation with an increased risk of prostate cancer.

A patient has just returned to the postsurgical unit from post-anesthetic recovery after breast surgery for removal of a malignancy. What is the most likely major nursing diagnosis to include in this patients immediate plan of care? A) Acute pain related to tissue manipulation and incision B) Ineffective coping related to surgery C) Risk for trauma related to post-surgical injury D) Chronic sorrow related to change in body image

A) Acute pain related to tissue manipulation and incision

A patient has been discharged home after a total mastectomy without reconstruction. The patient lives alone and has a home health referral. When the home care nurse performs the first scheduled visit this patient, what should the nurse assess? Select all that apply. A) Adherence to the exercise plan B) Overall psychological functioning C) Integrity of surgical drains d. Understanding of cancer e. Use of the breast prosthesis

A) Adherence to the exercise plan B) Overall psychological functioning C) Integrity of surgical drains

A patient comes to the free clinic complaining of a gray-white discharge that clings to her external vulva and vaginal walls. A nurse practitioner assesses the patient and diagnoses Gardnerella vaginalis. What would be the most appropriate nursing action at this time? A) Advise the patient that this is an overgrowth of normal vaginal flora. B) Discuss the effect of this diagnosis on the patients fertility. C) Document the vaginal discharge as normal. D) Administer acyclovir as ordered.

A) Advise the patient that this is an overgrowth of normal vaginal flora

A woman presents at the ED with sharp, colicky pain in her right abdomen that radiates to her right shoulder. She tells the nurse that she has been spotting lightly for the past few days. The patient is subsequently diagnosed with an ectopic pregnancy. What major nursing diagnosis most likely relates to this patients needs? A) Anxiety related to potential treatment options and health outcomes b. Chronic sorrow related to spontaneous abortion c. Chronic pain related to genitourinary trauma d. Impaired tissue integrity related to keloid scarring

A) Anxiety related to potential treatment options and health outcomes

A new mother who is breastfeeding calls the clinic to speak to a nurse. The patient is complaining of pain in her left breast and describes her breast as feeling doughy. The nurse tells her to come into the clinic and be checked. The patient is diagnosed with acute mastitis and placed on antibiotics. What comfort measure should the nurse recommend? A) Apply cold compresses as ordered. B) Avoid wearing a bra until the infection clears. C) Avoid washing the breasts. D) Perform gentle massage to stimulate neutrophil migration.

A) Apply cold compresses as ordered.

The nurse is performing the shift assessment of a postsurgical patient. The nurse finds his mental status, level of consciousness, speech, and orientation are intact and at baseline, but the patient tells you he is very anxious. What should the nurse do next? A) Assess the patients oxygen levels. B) Administer antianxiety medications. C) Page the patients the physician. D) Initiate a social work referral.

A) Assess the patients oxygen levels.

The perioperative nurse is providing care for a patient who is recovering on the postsurgical unit following a transurethral prostate resection (TUPR). The patient is reluctant to ambulate, citing the need to recover in bed. For what complication is the patient most at risk? A) Atelectasis B) Anemia C) Dehydration D) Peripheral edema

A) Atelectasis

A nurse is caring for a pregnant patient with active herpes. The teaching plan for this patient should include which of the following? A) Babies delivered vaginally may become infected with the virus. B) Recommended treatment is excision of the herpes lesions. C) Pain generally does not occur with a herpes outbreak during pregnancy. D) Pregnancy may exacerbate the mothers symptoms, but poses no risk to the infant.

A) Babies delivered vaginally may become infected with the virus.

A patient with trichomoniasis comes to the walk-in clinic. In developing a care plan for this patient the nurse would know to include what as an important aspect of treating this patient? A) Both partners will be treated with metronidazole (Flagyl). B) Constipation and menstrual difficulties may occur. C) The patient should perform Kegel exercises 30 to 80 times daily. D) Care will involve hormone therapy to control the pain.

A) Both partners will be treated with metronidazole (Flagyl).

A patient has just been told she needs to have an incisional biopsy of a right breast mass. During preoperative teaching, how could the nurse best assess this patient for specific educational, physical, or psychosocial needs she might have? A) By encouraging her to verbalize her questions and concerns B) By discussing the possible findings of the biopsy C) By discussing possible treatment options if the diagnosis is cancer D) By reviewing her medical history Ans: A

A) By encouraging her to verbalize her questions and concerns

A 35-year-old man is seen in the clinic because he is experiencing recurring episodes of urinary frequency, dysuria, and fever. The nurse should recognize the possibility of what health problem? A) Chronic bacterial prostatitis B) Orchitis C) Benign prostatic hyperplasia D)Urolithiasis

A) Chronic bacterial prostatitis

A nurse is providing care for a patient who has recently been admitted to the postsurgical unit from PACU following a transuretheral resection of the prostate. The nurse is aware of the nursing diagnosis of Risk for Imbalanced Fluid Volume. In order to assess for this risk, the nurse should prioritize what action? A) Closely monitoring the input and output of the bladder irrigation system B) Administering parenteral nutrition and fluids as ordered C) Monitoring the patients level of consciousness and skin turgor D) Scanning the patients bladder for retention every 2 hours

A) Closely monitoring the input and output of the bladder irrigation system

A 29-year-old patient has just been told that he has testicular cancer and needs to have surgery. During a presurgical appointment, the patient admits to feeling devastated that he requires surgery, stating that it will leave him emasculated and a shell of a man. The nurse should identify what nursing diagnosis when planning the patients subsequent care? A) Disturbed Body Image Related to Effects of Surgery B) Spiritual Distress Related to Effects of Cancer Surgery C) Social Isolation Related to Effects of Surgery D) Risk for Loneliness Related to Change in Self-Concept

A) Disturbed Body Image Related to Effects of Surgery

The nurse is assessing a 53-year-old woman who has been experiencing dysmenorrhea. What questions should the nurse include in an assessment of the patients menstrual history? Select all that apply. A) Do you ever experience bleeding after intercourse? B) How long is your typical cycle? C) Did you have any sexually transmitted infections in early adulthood? d. When did your mother and sisters get their first periods? E) Do you experience cramps or pain during your cycle

A) Do you ever experience bleeding after intercourse? B) How long is your typical cycle? E) Do you experience cramps or pain during your cycle

A nurse is teaching a 53-year-old man about prostate cancer. What information should the nurse provide to best facilitate the early identification of prostate cancer? A) Have a digital rectal examination and prostate-specific antigen (PSA) test done yearly. B) Have a transrectal ultrasound every 5 years. C) Perform monthly testicular self-examinations, especially after age 60. D) Have a complete blood count (CBC), blood urea nitrogen (BUN) and creatinine assessment performed annually.

A) Have a digital rectal examination and prostate-specific antigen (PSA) test done yearly.

A 21-year-old woman has sought care because of heavy periods and has subsequently been diagnosed with menorrhagia. The nurse should recognize which of the following as the most likely cause of the patients health problem? A) Hormonal disturbances B) Cervical or uterine cancer C) Pelvic inflammatory disease D) A sexually transmitted infection (STI)

A) Hormonal disturbances

The PACU nurse is caring for a 45-year-old male patient who had a left lobectomy. The nurse is assessing the patient frequently for airway patency and cardiovascular status. The nurse should know that the most common cardiovascular complications seen in the PACU include what? Select all that apply. A) Hypotension B) Hypervolemia C) Heart murmurs D) Dysrhythmias E) Hypertension

A) Hypotension D) Dysrhythmias E) Hypertension

A patient who is postoperative day 12 and recovering at home following a laparoscopic prostatectomy has reported that he is experiencing occasional dribbling of urine. How should the nurse best respond to this patients concern? A) Inform the patient that urinary control is likely to return gradually. B) Arrange for the patient to be assessed by his urologist. C) Facilitate the insertion of an indwelling urinary catheter by the home care nurse. D) Teach the patient to perform intermittent self-catheterization.

A) Inform the patient that urinary control is likely to return gradually.

An adolescent is brought to the clinic by her mother because of abnormal uterine bleeding. The nurse should understand that the most likely cause of this dysfunctional bleeding pattern is what? A) Lack of ovulation B) Chronic vaginitis C) A sexually transmitted infection D) Ectopic pregnancy

A) Lack of ovulation

The nurse is caring for a 63-year-old patient with ovarian cancer. The patient is to receive chemotherapy consisting of Taxol and Paraplatin. For what adverse effect of this treatment should the nurse monitor the patient? A) Leukopenia B) Metabolic acidosis C) Hyperphosphatemia D) Respiratory alkalosis

A) Leukopenia

The nurse is reviewing the physicians notes from the patient who has just left the clinic. The nurse learns that the physician suspects a malignant breast tumor. On palpation, the mass most likely had what characteristic? A) Nontenderness B) A size of 5 mm C) Softness and a regular shape D) Mobility

A) Nontenderness

The nurse is working with a patient who expects to begin menopause in the next few years. What educational topic should the nurse prioritize when caring for a healthy woman approaching menopause? A) Patient teaching and counseling regarding healthy lifestyles B) Referrals to local support groups C) Nutritional counseling regarding osteoporosis prevention D) Drug therapy options

A) Patient teaching and counseling regarding healthy lifestyles

A patient has experienced occasional urinary incontinence in the weeks since his prostatectomy. In order to promote continence, the nurse should encourage which of the following? A) Pelvic floor exercises B) Intermittent urinary catheterization C) Reduced physical activity D) Active range of motion exercises

A) Pelvic floor exercises

A 76-year-old with a diagnosis of penile cancer has been admitted to the medical floor. Because the incidence of penile cancer is so low, the staff educator has been asked to teach about penile cancer. What risk factors should the educator cite in this presentation? Select all that apply. A) Phimosis B) Priapism C) Herpes simplex infection D) Increasing age E) Lack of circumcision

A) Phimosis D) Increasing age E) Lack of circumcision

The nurse is planning the sexual assessment of a new adolescent patient. The nurse should include what assessment components? Select all that apply. A) Physical examination findings B) Laboratory results C) Health history D) Interpersonal skills E) Understanding of menopause

A) Physical examination findings B) Laboratory results C) Health history

A nurse is participating in a vaccination clinic at the local public health clinic. The nurse is describing the public health benefits of vaccinations to participants. Vaccine programs addressing which of the following diseases have been deemed successful? Select all that apply. A) Polio B) Diphtheria C) Hepatitis D) Tuberculosis E) Pertussis

A) Polio B) Diphtheria E) Pertussis

A nurse educator is reviewing the indications for chest drainage systems with a group of medical nurses. What indications should the nurses identify? Select all that apply. A) Post thoracotomy B) Spontaneous pneumothorax C) Need for postural drainage D) Chest trauma resulting in pneumothorax E) Pleurisy

A) Post thoracotomy B) Spontaneous pneumothorax D) Chest trauma resulting in pneumothorax

The nurse is caring for an 88-year-old patient who is recovering from an ileac-femoral bypass graft. The patient is day 2 postoperative and has been mentally intact, as per baseline. When the nurse assesses the patient, it is clear that he is confused and has been experiencing disturbed sleep patterns and impaired psychomotor skills. What should the nurse suspect is the problem with the patient? A) Postoperative delirium B) Postoperative dementia C) Senile dementia D) Senile confusion

A) Postoperative delirium

A public health nurse is participating in a campaign aimed at preventing cervical cancer. What strategies should the nurse include is this campaign? Select all that apply. A) Promotion of HPV immunization B) Encouraging young women to delay first intercourse C) Smoking cessation D) Vitamin D and calcium supplementation E) Using safer sex practices

A) Promotion of HPV immunization B) Encouraging young women to delay first intercourse C) Smoking cessation E) Using safer sex practices

A 22-year-old male is being discharged home after surgery for testicular cancer. The patient is scheduled to begin chemotherapy in 2 weeks. The patient tells the nurse that he doesnt think he can take weeks or months of chemotherapy, stating that he has researched the adverse effects online. What is the most appropriate nursing action for this patient at this time? A) Provide empathy and encouragement in an effort to foster a positive outlook. B) Tell the patient it is his decision whether to accept or reject chemotherapy. C) Report the patients statement to members of his support system. D) Refer the patient to social work.

A) Provide empathy and encouragement in an effort to foster a positive outlook.

A public health nurse is teaching a health class for the male students at the local high school. The nurse is teaching the boys to perform monthly testicular self-examinations. What point would be appropriate to emphasize? A) Testicular cancer is a highly curable type of cancer. B) Testicular cancer is very difficult to diagnose. C) Testicular cancer is the number one cause of cancer deaths in males. D) Testicular cancer is more common in older men.

A) Testicular cancer is a highly curable type of cancer.

By initiating an assessment about sexual concerns what does the nurse convey to the patient? Select all that apply. A) That sexual issues are valid health issues B) That it is safe to talk about sexual issues C) That sexual issues are only a minor aspect a persons identity D) That changes or problems in sexual functioning should be discussed E) That changes or problems in sexual functioning are highly atypical

A) That sexual issues are valid health issues B) That it is safe to talk about sexual issues D) That changes or problems in sexual functioning should be discussed

A gerontologic nurse is advocating for diagnostic testing of an 81-year-old patient who is experiencing personality changes. The nurse is aware of what factor that is known to affect the diagnosis and treatment of brain tumors in older adults? A) The effects of brain tumors are often attributed to the cognitive effects of aging. B) Brain tumors in older adults do not normally produce focal effects. C) Older adults typically have numerous benign brain tumors by the eighth decade of life. D) Brain tumors cannot normally be treated in patient over age 75.

A) The effects of brain tumors are often attributed to the cognitive effects of aging.

A 16-year-old male patient comes to the free clinic and is subsequently diagnosed with primary syphilis. What health problem most likely prompted the patient to seek care? A) The emergence of a chancre on his penis B) Painful urination C) Signs of a systemic infection D) Unilateral testicular swelling

A) The emergence of a chancre on his penis

The perioperative nurse is preparing to discharge a female patient home from day surgery performedunder general anesthetic. What instruction should the nurse give the patient prior to the patient leaving the hospital? A) The patient should not drive herself home. B) The patient should take an OTC sleeping pill for 2 nights. C) The patient should attempt to eat a large meal at home to aid wound healing. D) The patient should remain in bed for the first 48 hours postoperative.

A) The patient should not drive herself home.

An adult patient in the ICU has a central venous catheter in place. Over the past 24 hours, the patient has developed signs and symptoms that are suggestive of a central line associated bloodstream infection (CLABSI). What aspect of the patients care may have increased susceptibility to CLABSI? A) The patients central line was placed in the femoral vein. b)The patient had blood cultures drawn from the central line. c)The patient was treated for vancomycin-resistant enterococcus (VRE) during a previous admission. d)The patient has received antibiotics and IV fluids through the same line.

A) The patients central line was placed in the femoral vein.

A female patient with HIV has just been diagnosed with condylomata acuminata (genital warts). What information is most appropriate for the nurse to tell this patient? A) This condition puts her at a higher risk for cervical cancer; therefore, she should have a Papanicolaou (Pap) test annually. B) The most common treatment is metronidazole (Flagyl), which should eradicate the problem within 7 to 10 days. C) The potential for transmission to her sexual partner will be eliminated if condoms are used every time they have sexual intercourse. D) The human papillomavirus (HPV), which causes condylomata acuminata, cannot be transmitted during oral sex.

A) This condition puts her at a higher risk for cervical cancer; therefore, she should have a Papanicolaou (Pap) test annually.

You are caring for a patient who has been diagnosed with genital herpes. When preparing a teaching plan for this patient, what general guidelines should be taught? A) Thorough handwashing is essential. B) Sun bathing assists in eradicating the virus. C) Lesions should be massaged with ointment. D) Self-infection cannot occur from touching lesions during a breakout.

A) Thorough handwashing is essential.

The occupational health nurse is assessing new employees at a company. What would be important to assess in employees with a potential occupational respiratory exposure to a toxin? Select all that apply. A) Time frame of exposure B) Type of respiratory protection used C)Immunization status D)Breath sounds E)Intensity of exposure

A) Time frame of exposure B) Type of respiratory protection used D)Breath sounds E)Intensity of exposure

A female patient tells the nurse that she thinks she has a vaginal infection because she has noted inflammation of her vulva and the presence of a frothy, yellow-green discharge. The nurse recognizes that the clinical manifestations described are typical of what vaginal infection? A) Trichomonas vaginalis B) Candidiasis C) Gardnerella D) Gonorrhea Ans: A

A) Trichomonas vaginalis

A nurse is teaching a group of women about the potential benefits of breast self-examination (BSE). The nurse should teach the women that effective BSE is dependent on what factor? A) Womens knowledge of how their breasts normally look and feel B) The rapport that exists between the woman and her primary care provider C) Synchronizing womens routines around BSE with the performance of mammograms D) Womens knowledge of the pathophysiology of breast cancer

A) Womens knowledge of how their breasts normally look and feel

While planning a patients care, the nurse identifies nursing actions to minimize the patients pleuritic pain. Which intervention should the nurse include in the plan of care?

Avoid actions that will cause the patient to breathe deeply.

A patient comes to the clinic complaining of a tender, inflamed vulva. Testing does not reveal the presence of any known causative microorganism. What aspect of this patients current health status may account for the patients symptoms of vulvitis? A) The patient is morbidly obese. B) The patient has type 1 diabetes. C) The patient has chronic kidney disease. D) The patient has numerous allergies.

B) The patient has type 1 diabetes.

A patient has been diagnosed with endometriosis. When planning this patients care, the nurse should prioritize what nursing diagnosis? A) Anxiety related to risk of transmission B) Acute pain related to misplaced endometrial tissue C) Ineffective tissue perfusion related to hemorrhage D) Excess fluid volume related to abdominal distention

B) Acute pain related to misplaced endometrial tissue

A womans current health complaints are suggestive of a diagnosis of premenstrual dysphoric disorder (PMDD). The nurse should first do which of the following? A) Assess the patients understanding of HT. B) Assess the patient for risk of suicide. C) Assure the patient that the problem is self-limiting. D) Suggest the use of St. Johns wort.

B) Assess the patient for risk of suicide.

A 36-year-old woman comes to the clinic complaining of premenstrual syndrome (PMS) that is disrupting her quality of life. What signs and symptoms are associated with this health problem? Select all that apply. A) Loss of appetite B) Breast tenderness C) Depression D) Fluid retention E) Headache

B) Breast tenderness C) Depression D) Fluid retention E) Headache

The surgeons preoperative assessment of a patient has identified that the patient is at a high risk for venous thromboembolism. Once the patient is admitted to the postsurgical unit, what intervention should the nurse prioritize to reduce the patients risk of developing this complication? A) Maintain the head of the bed at 45 degrees or higher. B) Encourage early ambulation. C) Encourage oral fluid intake. D) Perform passive range-of-motion exercises every 8 hours.

B) Encourage early ambulation.

The nurse is caring for a couple trying to get pregnant and have not been able to for over a year. The couple asks what kind of problems a man can have that can cause infertility. What should be the nurses response? A) Men can have increased prolactin levels that decrease sperm viability. B) Men can have problems that increase the temperature around their testicles and decrease the quality of their semen. C) Men may inherit the gene that causes low sperm production. D) Men may produce sperm that are incompatible with the shape of the egg.

B) Men can have problems that increase the temperature around their testicles and decrease the quality of their semen.

A patient is 2 hours postoperative with a Foley catheter in situ. The last hourly urine output recorded for this patient was 10 mL. The tubing of the Foley is patent. What should the nurse do? A) Irrigate the Foley with 30 mL normal saline. B) Notify the physician and continue to monitor the hourly urine output closely. C) Decrease the IV fluid rate and massage the patients abdomen. D) Have the patient sit in high-Fowlers position.

B) Notify the physician and continue to monitor the hourly urine output closely.

The nurse is assessing a patient who frequently coughs after eating or drinking. How should the nurse best follow up this assessment finding? A) Obtain a sputum sample. B) Perform a swallowing assessment. C) Inspect the patients tongue and mouth. D) Assess the patients nutritional status.

B) Perform a swallowing assessment.

A patient has been diagnosed with pulmonary hypertension, in which the capillaries in the alveoli are squeezed excessively. The nurse should recognize a disturbance in what aspect of normal respiratory function? A) Acidbase balance B) Perfusion C) Diffusion D) V entilation

B) Perfusion

The nurse is caring for a patient who is postoperative day 2 following a colon resection. While turning him, wound dehiscence with evisceration occurs. What should be the nurses first response? A) Return the patient to his previous position and call the physician. B) Place saline-soaked sterile dressings on the wound. C) Assess the patients blood pressure and pulse. D) Pull the dehiscence closed using gloved hands.

B) Place saline-soaked sterile dressings on the wound.

A 52-year-old woman has just been told she has breast cancer and is scheduled for a modified mastectomy the following week. The nurse caring for this patient knows that she is anxious and fearful about the upcoming procedure and the newly diagnosed malignancy. How can the nurse most likely alleviate this patients fears? A) Provide written material on the procedure that has been scheduled for the patient. B) Provide the patient with relevant information about expected recovery. C) Give the patient current information on breast cancer survival rates. D) Offer the patient alternative treatment options.

B) Provide the patient with relevant information about expected recovery.

A postoperative patient rapidly presents with hypotension; rapid, thready pulse; oliguria; and cold, pale skin. The nurse suspects that the patient is experiencing a hemorrhage. What should be the nurses first action? A) Leave and promptly notify the physician. B) Quickly attempt to determine the cause of hemorrhage. C) Begin resuscitation. D) Put the patient in the Trendelenberg position.

B) Quickly attempt to determine the cause of hemorrhage.

A man comes to the clinic complaining that he is having difficulty obtaining an erection. When reviewing the patients history, what might the nurse note that contributes to erectile dysfunction? A) The patient has been treated for a UTI twice in the past year. B) The patient has a history of hypertension. C) The patient is 66 years old. D) The patient leads a sedentary lifestyle.

B) The patient has a history of hypertension.

A 35-year-old father of three tells the nurse that he wants information on a vasectomy. What would the nurse tell him about ejaculate after a vasectomy? A) There will be no ejaculate after a vasectomy, though the patients potential for orgasm is unaffected. B) There is no noticeable decrease in the amount of ejaculate even though it contains no sperm. C) There is a marked decrease in the amount of ejaculate after vasectomy, though this does not affect sexual satisfaction. D) There is no change in the quantity of ejaculate after vasectomy, but the viscosity is somewhat increased.

B) There is no noticeable decrease in the amount of ejaculate even though it contains no sperm.

A patient with a decreased level of consciousness is in a recumbent position. How should the nurse best assess the lung fields for a patient in this position? A) Inform that physician that the patient is in a recumbent position and anticipate an order for a portable chest x-ray. B) Turn the patient to enable assessment of all the patients lung fields. C) Avoid turning the patient, and assess the accessible breath sounds from the anterior chest wall. D) Obtain a pulse oximetry reading, and, if the reading is low, reposition the patient and auscultate breath sounds.

B) Turn the patient to enable assessment of all the patients lung fields.

An older adult patient tells the nurse that she had chicken pox as a child and is eager to be vaccinated against shingles. What should the nurse teach the patient about this vaccine? A) Vaccination against shingles is contraindicated in patients over the age of 80. B) Vaccination can reduce her risk of shingles by approximately 50%. C) Vaccination against shingles involves a series of three injections over the course of 6 months. D) Vaccination against shingles is only effective if preceded by a childhood varicella vaccination.

B) Vaccination can reduce her risk of shingles by approximately 50%.

A 14-year-old is brought to the clinic by her mother. The mother explains to the nurse that her daughter has just started using tampons, but is not yet sexually active. The mother states I am very concerned because my daughter is having a lot of stabbing pain and burning. What might the nurse suspect is the problem with the 14-year-old? A) Vulvitis B) Vulvodynia C) V aginitis D) Bartholins cyst

B) Vulvodynia

The nurse educator is discussing neoplasms with a group of recent graduates. The educator explains that the effects of neoplasms are caused by the compression and infiltration of normal tissue. The physiologic changes that result can cause what pathophysiologic events? Select all that apply. A) Intracranial hemorrhage B) Infection of cerebrospinal fluid C) Increased ICP D) Focal neurologic signs E) Altered pituitary function

C) Increased ICP D) Focal neurologic signs E) Altered pituitary function

The nurse leading an educational session is describing self-examination of the breast. The nurse tells the womens group to raise their arms and inspect their breasts in a mirror. A member of the womens group asks the nurse why raising her arms is necessary. What is the nurses best response? A) It helps to spread out the fat that makes up your breast. B) It allows you to simultaneously assess for pain. C) It will help to observe for dimpling more closely. D) This is what the American Cancer Society recommends.

C) It will help to observe for dimpling more closely.

The intraoperative nurse is transferring a patient from the OR to the PACU after replacement of the right knee. The patient is a 73-year-old woman. The nurse should prioritize which of the following actions? A) Keeping the patient sterile B) Keeping the patient restrained C) Keeping the patient warm D) Keeping the patient hydrated

C) Keeping the patient warm

A nurse is explaining that each breast contains 12 to 20 cone-shaped lobes. The nurse should explain that each lobe consists of what elements? A) Modified tendons and ligaments B) Connective tissue and smooth muscle C) Lobules and ducts D) Endocrine glands and sebaceous glands

C) Lobules and ducts

A clinic nurse is caring for a male patient diagnosed with gonorrhea who has been prescribed ceftriaxone and doxycycline. The patient asks why he is receiving two antibiotics. What is the nurses best response? A) There are many drug-resistant strains of gonorrhea, so more than one antibiotic may be required for successful treatment. B) The combination of these two antibiotics reduces the later risk of reinfection. C) Many people infected with gonorrhea are infected with chlamydia as well. D) This combination of medications will eradicate the infection twice as fast than a single antibiotic.

C) Many people infected with gonorrhea are infected with chlamydia as well.

The nurse is reviewing the electronic health record of a patient with an empyema. What health problem in the patients history is most likely to have caused the empyema? A) Smoking B) Asbestosis C) Pneumonia D) Lung cancer

C) Pneumonia

A nurse practitioner is assessing a 55-year-old male patient who is complaining of perineal discomfort, burning, urgency, and frequency with urination. The patient states that he has pain with ejaculation. The nurse knows that the patient is exhibiting symptoms of what? A) V aricocele B) Epididymitis C) Prostatitis D) Hydrocele

C) Prostatitis

A patient diagnosed with cervical cancer will soon begin a round of radiation therapy. When planning the patients subsequent care, the nurse should prioritize actions with what goal? A) Preventing hemorrhage B) Ensuring the patient knows the treatment is palliative, not curative C) Protecting the safety of the patient, family, and staff D) Ensuring that the patient adheres to dietary restrictions during treatment

C) Protecting the safety of the patient, family, and staff

The nurse is caring for a patient on the medicalsurgical unit postoperative day 5. During each patient assessment, the nurse evaluates the patient for infection. Which of the following would be most indicative of infection? A) Presence of an indwelling urinary catheter B) Rectal temperature of 99.5F (37.5C) C) Red, warm, tender incision D) White blood cell (WBC) count of 8,000/mL

C) Red, warm, tender incision

A 57-year-old male comes to the clinic complaining that when he has an erection his penis curves and becomes painful. The patients diagnosis is identified as severe Peyronies disease. The nurse should be aware of what likely treatment modality? A) Physical therapy B) Treatment with PDE-5 inhibitors C) Intracapsular hydrocortisone injections D) Surgery

D) Surgery

A clinic nurse is meeting with a 38-year-old patient who states that she would like to resume using oral contraceptives, which she used for several years during her twenties. What assessment question is most likely to reveal a potential contraindication to oral contraceptive use? A) Have you ever had surgery? B) Have you ever had a sexually transmitted infection? C) When did you last have your blood sugar levels checked? D) Do you smoke?

D) Do you smoke?

The nurse is admitting a patient to the medicalsurgical unit from the PACU. What should the nurse do to help the patient clear secretions and help prevent pneumonia? A) Encourage the patient to eat a balanced diet that is high in protein. B) Encourage the patient to limit his activity for the first 72 hours. C) Encourage the patient to take his medications as ordered. D) Encourage the patient to use the incentive spirometer every 2 hours.

D) Encourage the patient to use the incentive spirometer every 2 hours.

A patients total laryngectomy has created a need for alaryngeal speech which will be achieved through the use of tracheoesophageal puncture. What action should the nurse describe to the patient when teaching him about this process? A) Training on how to perform controlled belching B) Use of an electronically enhanced artificial pharynx C) Insertion of a specialized nasogastric tube D) Fitting for a voice prosthesis

D) Fitting for a voice prosthesis

The nurses aide notifies the nurse that a patient has decreased oxygen saturation levels. The nurseassesses the patient and finds that he is tachypnic, has crackles on auscultation, and his sputum is frothy and pink. The nurse should suspect what complication? A) Pulmonary embolism B) Atelectasis C) Laryngospasm D) Flash pulmonary edema

D) Flash pulmonary edema

A patient has presented at the clinic with symptoms of benign prostatic hyperplasia. What diagnostic findings would suggest that this patient has chronic urinary retention? A) Hypertension B) Peripheral edema C) Tachycardia and other dysrhythmias D) Increased blood urea nitrogen (BUN)

D) Increased blood urea nitrogen (BUN)

A public health nurse has been asked to provide a health promotion session for men at a wellness center. What should the nurse inform the participants about testicular cancer? A) It is most common among men over 55. B) It is one of the least curable solid tumors. C) It typically does not metastasize. D) It is highly responsive to treatment.

D) It is highly responsive to treatment.

A 27-year-old primipara presents to the ED with vaginal bleeding and suspected contractions. The woman relates that she is 14 weeks pregnant and she thinks she is losing her baby. Diagnostic testing confirms a spontaneous abortion. What nursing action would be a priority at this time? A) Leave the patient alone so she can grieve in private. B) Teach the patient that this will not affect her future chance of conception. C) Take the patient off the obstetric floor so she will not hear a baby cry. D) Provide opportunities for the patient to talk and express her emotions.

D) Provide opportunities for the patient to talk and express her emotions.

A nurse has assessed that a patient is not yet willing to view her mastectomy site. How should the nurse best assist the patient is developing a positive body image? A) Ask the woman to describe the current appearance of her breast. B) Help the patient to understand that many women have gone through the same unpleasant experience. C) Explain to the patient that her body image does not have to depend on her physical appearance. D) Provide the patient with encouragement in an empathic and thoughtful manner.

D) Provide the patient with encouragement in an empathic and thoughtful manner.

A patient has presented for her annual mammogram. The patient voices concerns related to exposure to radiation. What should the nurse teach the patient about a mammogram? A) It does not use radiation. B) Radiation levels are safe as long as mammograms are performed only once per year. C) The negative effects of radiation do not accumulate until late in life. D) Radiation from a mammogram is equivalent to an hour of sunlight.

D) Radiation from a mammogram is equivalent to an hour of sunlight.

A 27-year-old female patient is diagnosed with invasive cervical cancer and is told she needs to have a hysterectomy. One of the nursing diagnoses for this patient is disturbed body image related to perception of femininity. What intervention would be most appropriate for this patient? A) Reassure the patient that she will still be able to have children. B) Reassure the patient that she does not have to have sex to be feminine. C) Reassure the patient that you know how she is feeling and that you feel her anxiety and pain. D) Reassure the patient that she will still be able to have intercourse with sexual satisfaction and orgasm.

D) Reassure the patient that she will still be able to have intercourse with sexual satisfaction and orgasm.

The nurse is working with a couple who is being evaluated for infertility. What nursing intervention would be most appropriate for this couples likely needs? A) Educating them about parenting techniques in order to foster hope B) Educating them about the benefits of child-free living C) Choosing the most appropriate reproductive technology D) Referring them to appropriate community resources

D) Referring them to appropriate community resources

The home care nurse is monitoring a patient discharged home after resolution of a pulmonary embolus. For what potential complication would the home care nurse be most closely monitoring this patient? A) Signs and symptoms of pulmonary infection B) Swallowing ability and signs of aspiration C) Activity level and role performance D) Residual effects of compromised oxygenation

D) Residual effects of compromised oxygenation

The nurse is caring for a patient who has just returned to the unit after a colon resection. The patient is showing signs of hypoxia. The nurse knows that this is probably caused by what? A) Nitrogen narcosis B) Infection C) Impaired diffusion D) Shunting

D) Shunting

The nurse is caring for a 52-year-old woman whose aunt and mother died of breast cancer. The patient states, My doctor and I talked about Tamoxifen to help prevent breast cancer. Do you think it will work?What would be the nurses best response? A) Yes, its known to have a slight protective effect. B) Yes, but studies also show an increased risk of osteoporosis. C) You wont need to worry about getting cancer as long as you take Tamoxifen. D) Tamoxifen is known to be a highly effective protective measure.

D) Tamoxifen is known to be a highly effective protective measure.

A 17-year-old girl has come to the free clinic for her annual examination. She tells the nurse she uses tampons and asks how long she may safely leave her tampon in place. What is the nurses best response? A) You may leave the tampon in overnight. B) The tampon should be changed at least twice per day. C) Tampons are dangerous and, ideally, you should not be using them. D) Tampons need to be changed every 4 to 6 hours.

D) Tampons need to be changed every 4 to 6 hours.

The nurse is providing preoperative education for a patient diagnosed with endometriosis. A hysterectomy has been scheduled. What education topic should the nurse be sure to include for this patient? A) Menstrual periods will continue to occur for several months, some of them heavy. B) Normal activity will be permitted within 48 hours following surgery. C) After a hysterectomy, hormone levels remain largely unaffected. D) The bladder must be emptied prior to surgery and a catheter may be placed during surgery.

D) The bladder must be emptied prior to surgery and a catheter may be placed during surgery.

The decision has been made to discharge a ventilator-dependent patient home. The nurse is developing a teaching plan for this patient and his family. What would be most important to include in this teaching plan? Administration of inhaled corticosteroids Assessment of neurologic status Turning and coughing Signs of pulmonary infection

Signs of pulmonary infection

A physician explains to the patient that he has an inflammation of the Cowper glands. Where are the Cowper glands located? A) Within the epididymis b. Below the prostate, within the posterior aspect of the urethra c. On the inner epithelium lining the scrotum, lateral to the testes d. Medial to the vas deferens

b. Below the prostate, within the posterior aspect of the urethra

A patient presents to the emergency department with paraphimosis. The physician is able to compress the glans and manually reduce the edema. Once the inflammation and edema subside, what is usually indicated? a. Needle aspiration of the corpus cavernosum b. Circumcision c. Abstinence from sexual activity for 6 weeks d. Administration of vardenafil

b. Circumcision

A patient who came to the clinic after finding a mass in her breast is scheduled for a diagnostic breast biopsy. During the nurses admission assessment, the nurse observes that the patient is distracted and tense. What is it important for the nurse to do? a. Acknowledge the fear the patient is likely experiencing. b. Describe the support groups that exist in the community. c. Assess the patients stress management skills. d. Document a nursing diagnosis of ineffective coping.

a. Acknowledge the fear the patient is likely experiencing.

The nurse is teaching a transgender client about the medication goserelin (Zoladex). What action by the client indicates good understanding? a. Takes a manual blood pressure b. Administers a subcutaneous injection c. Prepares an implanted port for IV insertion d. States that the axillary area will be clothed

b. Administers a subcutaneous injection

A patient is to undergo an ultrasound-guided core biopsy. The patient tells the nurse that a friend of hers had a stereotactic core biopsy. She wants to understand the differences between the two procedures. What would be the nurses best response? a. An ultrasound-guided core biopsy is faster, less expensive, and does not use radiation. b. An ultrasound-guided core biopsy is a little more expensive, but it doesnt use radiation and it is faster. c. An ultrasound-guided core biopsy is a little more expensive, and it also uses radiation but it is faster. d. An ultrasound-guided core biopsy takes more time, and it also uses radiation, but it is lessexpensive.

a. An ultrasound-guided core biopsy is faster, less expensive, and does not use radiation.

A client has returned from the postanesthesia care unit after a vaginoplasty. What comfort measure does the nurse provide for this client? a. Apply ice to the perineum. b. Elevate the legs on pillows. c. Position the client on the left side. d. Raise the head of the bed.

a. Apply ice to the perineum.

A nurse is providing health teaching to a middle-aged male-to-female (MtF) client who has undergone gender reassignment surgery. What information is most important to this client? a. Be sure to have an annual prostate examination. b. Continue your normal health screenings. c. Try to avoid being around people who are ill. d. You should have an annual flu vaccination.

a. Be sure to have an annual prostate examination.

A nurse practitioner is examining a patient who presented at the free clinic with vulvar pruritus. For which assessment finding would the practitioner look that may indicate the patient has an infection caused by Candida albicans? a. Cottage cheese-like discharge b. Yellow-green discharge c. Gray-white discharge d. Watery discharge with a fishy odor

a. Cottage cheese-like discharge

An adult patient has just been admitted to the PACU following abdominal surgery. As the patient begins to awaken, he is uncharacteristically restless. The nurse checks his skin and it is cold, moist, and pale. The nurse concerned the patient may be at risk for what? a. Hemorrhage and shock b. Aspiration c. Postoperative infection d. Hypertension and dysrhythmias

a. Hemorrhage and shock

1. The nurse is reviewing possible complications from a phalloplasty. What factors does the nurse include? (Select all that apply.) a. Infection of donor site b. Necrosis of the neopenis c. Rectal perforation d. Urinary tract stenosis e. Vaginal infections

a. Infection of donor site b. Necrosis of the neopenis d. Urinary tract stenosis

A patient has been referred to the breast clinic after her most recent mammogram revealed the presence of a lump. The lump is found to be a small, well-defined nodule in the right breast. The oncology nurse should recognize the likelihood of what treatment? a. Lumpectomy and radiation b. Partial mastectomy and radiation c. Partial mastectomy and chemotherapy d. Total mastectomy and chemotherapy

a. Lumpectomy and radiation

A 15-year-old girl is brought to the clinic by her mother to see her primary care provider. The mother states that her daughter has not started to develop sexually. The physical examination shows that the patient has no indication of secondary sexual characteristics. What diagnosis should the nurse suspect? a. Primary amenorrhea b. Dyspareunia c. Vaginal atrophy d. Secondary dysmenorrhea

a. Primary amenorrhea

The nurse is caring for a patient who has just been told that her ovarian cancer is terminal and that no curative options remain. What would be the priority nursing care for this patient at this time? a. Provide emotional support to the patient and her family. b. Implement distraction and relaxation techniques. c. Offer to inform the patients family of this diagnosis. d. Teach the patient about the importance of maintaining a positive attitude.

a. Provide emotional support to the patient and her family.

Following a recent history of dyspareunia and lower abdominal pain, a patient has received a diagnosis of pelvic inflammatory disease (PID). When providing health education related to self-care, the nurse should address which of the following topics? Select all that apply. a. Use of condoms to prevent infecting others b. Appropriate use of antibiotics c. Taking measures to prevent pregnancy d. The need for a Pap smear every 3 months d. The importance of weight loss in preventing symptoms

a. Use of condoms to prevent infecting others b. Appropriate use of antibiotics

A patient who involved in a workplace accident suffered a penetrating wound of the chest that led to acute respiratory failure. What goal of treatment should the care team prioritize when planning this patients care? a)Facilitation of long-term intubation b)Restoration of adequate gas exchange c)Attainment of effective coping D) Self-management of oxygen therapy

b)Restoration of adequate gas exchange

The nurse educator is discussing emerging diseases with a group of nurses. The educator should cite what causes of emerging diseases? Select all that apply. a)Progressive weakening of human immune systems b)Use of extended-spectrum antibiotics c)Population movements d)Increased global travel e)Globalization of food supplies

b)Use of extended-spectrum antibiotics c)Population movements d)Increased global travel e)Globalization of food supplies

The nurse admits a patient to the PACU with a blood pressure of 132/90 mm Hg and a pulse of 68 beats per minute. After 30 minutes, the patients blood pressure is 94/47 mm Hg, and the pulse is 110. The nurse documents that the patients skin is cold, moist, and pale. Of what is the patient showing signs? a. Hypothermia b. Hypovolemic shock c. Neurogenic shock d. Malignant hyperthermia

b. Hypovolemic shock

A client had a vaginoplasty under epidural anesthetic. Which action by the nurse is most important? a. Ensure that the urinary catheter is securely attached to the leg. b. Instruct the client not to try to get out of bed unassisted. c. Monitor the clients dressings and wound drainage. d. Position the Jackson-Pratt drain to the contralateral side.

b. Instruct the client not to try to get out of bed unassisted.

A patient with a genital herpes exacerbation has a nursing diagnosis of acute pain related to the genital lesions. What nursing intervention best addresses this diagnosis? a. Cover the lesions with a topical antibiotic. b. Keep the lesions clean and dry. c. Apply a topical NSAID to the lesions. d. Remain on bed rest until the lesions resolve.

b. Keep the lesions clean and dry.

The nurse is caring for a patient who has just been transferred to the PACU from the OR. What is the highest nursing priority? a. Assessing for hemorrhage b. Maintaining a patent airway c. Managing the patients pain d. Assessing vital signs every 30 minutes

b. Maintaining a patent airway

A 60-year-old man presents at the clinic complaining that his breasts are tender and enlarging. The patient is subsequently diagnosed with gynecomastia. The patient should be assessed for the possibility of what causative factor? A) Age-related physiologic changes b. Medication adverse effects c. Poor nutrition d. Fluid overload

b. Medication adverse effects

A patient is being discharged home from the ambulatory surgery center after an incisional biopsy of a mass in her left breast. What are the criteria for discharging this patient home? Select all that apply. a. Patient must understand when she can begin ambulating b. Patient must have someone to accompany her home c. Patient must understand activity restrictions d. Patient must understand care of the biopsy site e. Patient must understand when she can safely remove her urinary catheter

b. Patient must have someone to accompany her home c. Patient must understand activity restrictions d. Patient must understand care of the biopsy site

A client is preparing for gender reassignment surgery and will transition from male to female. The client is worried about the voice not sounding feminine enough. What action by the nurse is best? a. Ask if the client has considered vocal cord surgery to change the voice. b. Refer the client for vocal therapy with speech-language pathology. c. Teach the client that there will be no effect on the clients voice. d. Tell the client that the use of hormones will eventually change the voice.

b. Refer the client for vocal therapy with speech-language pathology.

The nursing instructor is discussing postoperative care with a group of nursing students. A student nurse asks, Why does the patient go to the PACU instead of just going straight up to the postsurgical unit? What is the nursing instructors best response? a. The PACU allows the patient to recover from anesthesia in a stimulating environment to facilitate awakening and reorientation. b. The PACU allows the patient to recover from the effects of anesthesia, and the patient stays in the PACU until he or she is oriented, has stable vital signs, and is without complications. c. Frequently, patients are placed in the medicalsurgical unit to recover, but hospitals are usually short of beds, and the PACU is an excellent place to triage patients. d. Patients remain in the PACU for a predetermined time because the surgeon will often need to reinforce or alter the patients incision in the hours following surgery.

b. The PACU allows the patient to recover from the effects of anesthesia, and the patient stays in the PACU until he or she is oriented, has stable vital signs, and is without complications.

The nurse is caring for a postoperative patient who needs daily dressing changes. The patient is 3 days postoperative and is scheduled for discharge the next day. Until now, the patient has refused to learn how to change her dressing. What would indicate to the nurse the patients possible readiness to learn how to change her dressing? Select all that apply. a. The patient wants you to teach a family member to do dressing changes. b. The patient expresses interest in the dressing change. c. The patient is willing to look at the incision during a dressing change. d. The patient expresses dislike of the surgical wound. e. The patient assists in opening the packages of dressing material for the nurse.

b. The patient expresses interest in the dressing change. c. The patient is willing to look at the incision during a dressing change. e. The patient assists in opening the packages of dressing material for the nurse.

A patient underwent an open bowel resection 2 days ago and the nurses most recent assessment of the patients abdominal incision reveals that it is dehiscing. What factor should the nurse suspect may have caused the dehiscence? a. The patients surgical dressing was changed yesterday and today. b. The patient has vomited three times in the past 12 hours. c. The patient has begun voiding on the commode instead of a bedpan. d. The patient used PCA until this morning.

b. The patient has vomited three times in the past 12 hours.

A middle-aged female patient has been offered testing for HIV/AIDS upon admission to the hospital for an unrelated health problem. The nurse observes that the patient is visibly surprised and embarrassed by this offer. How should the nurse best respond? a. Most women with HIV dont know they have the disease. If you have it, its important we catch it early. b. This testing is offered to every adolescent and adult regardless of their lifestyle, appearance or history. c. The rationale for this testing is so that you can begin treatment as soon as testing comes back, if its positive. d. Youre being offered this testing because you are actually in the prime demographic for HIV infection.

b. This testing is offered to every adolescent and adult regardless of their lifestyle, appearance or history.

A nurse is performing an admission assessment on a 40-year-old man who has been admitted for outpatient surgery on his right knee. While taking the patients family history, he states, My father died of prostate cancer at age 48. The nurse should instruct him on which of the following health promotion activities? a. The patient will need PSA levels drawn starting at age 55. b. The patient should have testing for presence of the CDH1 and STK11 genes. c. The patient should have PSA levels drawn regularly. D) The patient should limit alcohol use due to the risk of malignancy.

c. The patient should have PSA levels drawn regularly.

The PACU nurse is caring for a patient who has arrived from the OR. During the initial assessment, the nurse observes that the patients skin has become blue and dusky. The nurse looks, listens, and feels for breathing, and determines the patient is not breathing. What is the priority intervention? a. Check the patients oxygen saturation level, continue to monitor for apnea, and perform a focused assessment. b. Treat the possible airway obstruction by tilting the head back and pushing forward on the angle ofthe lower jaw. c. Assess the arterial pulses, and place the patient in the Trendelenburg position. d. Reintubate the patient.

b. Treat the possible airway obstruction by tilting the head back and pushing forward on the angle ofthe lower jaw.

A patient calls the clinic and tells the nurse she has thick white, curd-like discharge from her vagina. How should the nurse best interpret this preliminary data? a.The drainage is physiologic and normal. b.The patient may have a Candida species infection. c.The patient needs a Pap smear as soon as possible. d.The patient may have a Trichomonas infection.

b.The patient may have a Candida species infection.

The nurse in the ED is caring for a man who has returned to the ED 4 days after receiving stitches for a knife wound on his hand. The wound is now infected, so the stitches were removed, and the wound is cleaned and packed with gauze. The ED doctor plans to have the man return tomorrow to remove the packing and resuture the wound. You are aware that the wound will now heal by what means? a. Late intention b. Second intention c. Third intention d. First intention

c. Third intention

A nurse providing prenatal care to a pregnant woman is addressing measures to reduce her postpartum risk of cystocele, rectocele, and uterine prolapse. What action should the nurse recommend? a. Maintenance of good perineal hygiene b. Prevention of constipation c. Increased fluid intake for 2 weeks postpartum d. Performance of pelvic muscle exercises

d. Performance of pelvic muscle exercises

A nurse is reviewing the chart of a new client in the family medicine clinic and notes the client is identified as George Smith. The nurse enters the room and finds a woman in a skirt. What action by the nurse is best? a. Apologize and declare confusion about the client. b. Ask Mrs. Smith where her husband is right now. c. Ask the client about preferred forms of address. d. Explain that the chart must contain an error.

c. Ask the client about preferred forms of address.

A nursing student is learning how to perform sexual assessments using the PLISSIT model. According to this model, the student should begin an assessment by doing which of the following? a. Briefly teaching the patient about normal sexual physiology b. Assuring the patient that what he says will be confidential c. Asking the patient if he is willing to discuss sexual functioning d. Ensuring patient privacy

c. Asking the patient if he is willing to discuss sexual functioning

The nurse is planning health education for a patient who has experienced a vaginal infection. What guidelines should the nurse include in this program regarding prevention? a. Wear tight-fitting synthetic underwear. b. Use bubble bath to eradicate perineal bacteria. c. Avoid feminine hygiene products, such as sprays. d. Restrict daily bathing.

c. Avoid feminine hygiene products, such as sprays.

A 45-year-old woman has just undergone a radical hysterectomy for invasive cervical cancer. Prior to the surgery the physician explained to the patient that after the surgery a source of radiation would be placed near the tumor site to aid in reducing recurrence. What is the placement of the source of radiation called? A) Internal beam radiation b. Trachelectomy c. Brachytherapy . d. External radiation

c. Brachytherapy

The home health nurse is caring for a postoperative patient who was discharged home on day 2 after surgery. The nurse is performing the initial visit on the patients postoperatative day 2. During the visit, the nurse will assess for wound infection. For most patients, what is the earliest postoperative day that a wound infection becomes evident? a. Day 9 b. Day 7 c. Day 5 d. Day 3

c. Day 5

A nurse is caring for a 33-year-old male who has come to the clinic for a physical examination. He states that he has not had a routine physical in 5 years. During the examination, the physician finds that digital rectal examination (DRE) reveals stoney hardening in the posterior lobe of the prostate gland that is not mobile. The nurse recognizes that the observation typically indicates what? a. A normal finding b. A sign of early prostate cancer c. Evidence of a more advanced lesion d. Metastatic disease

c. Evidence of a more advanced lesion

The nurse is caring for an 82-year-old female patient in the PACU. The woman begins to awaken and responds to her name, but is confused, restless, and agitated. What principle should guide the nurses subsequent assessment? a. Postoperative confusion in older adults is an indication of impaired oxygenation or possibly a stroke during surgery. b. Confusion, restlessness, and agitation are expected postoperative findings in older adults and they will diminish in time. c. Postoperative confusion is common in the older adult patent, but it could also indicate a significant blood loss. d. Confusion, restlessness, and agitation indicate an underlying cognitive deficit such as dementia.

c. Postoperative confusion is common in the older adult patent, but it could also indicate a significant blood loss.

A 51-year-old woman is experiencing perimenopausal symptoms and expresses confusion around the possible use of hormone therapy (HT). She explains that her mother and aunts used HT and she is unsure why few of her peers have been prescribed this treatment. What should the nurse explain to the patient? a. Large, long-term health studies have revealed that HT is minimally effective. b. HT has been largely replaced by other nonpharmacologic interventions. c. Research has shown that significant health risks are associated with HT. d. HT has been shown to exacerbate symptoms of menopause in a minority of women.

c. Research has shown that significant health risks are associated with HT.

The nurse is caring for a patient after abdominal surgery in the PACU. The patients blood pressure has increased and the patient is restless. The patients oxygen saturation is 97%. What cause for this change in status should the nurse first suspect? a. The patient is hypothermic. b. The patient is in shock. c. The patient is in pain. d. The patient is hypoxic.

c. The patient is in pain.

A woman calls the clinic and tells the nurse she has had bloody drainage from her right nipple. The nurse makes an appointment for this patient, expecting the physician or practitioner to order what diagnostic test on this patient? A) Breast ultrasound B) Radiography c. Positron emission testing (PET) d. Galactography

d. Galactography

The dressing surrounding a mastectomy patients Jackson-Pratt drain has scant drainage on it. The nurse believes that the amount of drainage on the dressing may be increasing. How can the nurse best confirm this suspicion? a. Describe the appearance of the dressing in the electronic health record. b. Photograph the patients abdomen for later comparison using a smartphone. c. Trace the outline of the drainage on the dressing for future comparison. d. Remove and weigh the dressing, reapply it, and then repeat in 8 hours.

c. Trace the outline of the drainage on the dressing for future comparison.

A patient has been diagnosed with polycystic ovary syndrome (PCOS). The nurse should encourage what health promotion activity to address the patients hormone imbalance and infertility? a. Kegel exercises b. Increased fluid intake c. Weight loss d. Topical antibiotics as ordered

c. Weight loss

The nurse is caring for a postoperative client following an anterior colporrhaphy. What action can be delegated to the unlicensed assistive personnel (UAP)? a. Reviewing the hematocrit and hemoglobin results b. Teaching the client to avoid lifting her 4-year-old grandson c. Assessing the level of pain and any drainage d. Drawing a shallow hot bath for comfort measures

d. Drawing a shallow hot bath for comfort measures

During the nurses assessment of a female patient, the patient reveals that she experienced sexual abuse when she was a young woman. What is the nurses most appropriate response to this disclosure? a. Reassure her that this information will be kept a secret. b. Begin the process of intensive psychotherapy. c. Encourage the patient to phone 911. d. Facilitate appropriate resources and referrals.

d. Facilitate appropriate resources and referrals.


Set pelajaran terkait

Adam Smith, father of modern economics

View Set

Stages in the Healing of a Bone Fracture

View Set

Peak Test Questions And Vocabulary Review

View Set

Contracts - E&E - The Parol Evidence Rule

View Set

NCLEX cardiovascular, hematologic, and lymphatic systems

View Set

IT Final - Chapters 5-10, 13 and 14

View Set

Vegetable Crop Production Evaluation II

View Set